หัวข้อ: Road to IMO 2017 to Infinity
ดูหนึ่งข้อความ
  #9  
Old 20 ตุลาคม 2016, 10:48
Beatmania's Avatar
Beatmania Beatmania ไม่อยู่ในระบบ
ลมปราณคุ้มครองร่าง
 
วันที่สมัครสมาชิก: 10 พฤษภาคม 2011
ข้อความ: 279
Beatmania is on a distinguished road
Default

ว่าแล้วว่าต้องเป็นน้องในค่าย 5555 จะจัดให้แบบไม่ผิดหวังแน่นอนครับๆ เอา I ไปก่อนแล้วกัน

1. ให้ $x_1\leq x_2\leq ...\leq x_n$ เป็นจำนวนจริงบวกโดยที่ $\frac{x_1}{1}\geq \frac{x_2}{2}\geq ... \geq \frac{x_n}{n}$ จงแสดงว่า

$$\frac{x_1+x_2+...+x_n}{n\sqrt[n]{x_1x_2...x_n} }\leq \frac{n+1}{2\sqrt[n]{n!} }$$

(China TST 2015)

2. ให้ $a_1,a_2,...,a_n$ เป็นจำนวนจริงบวก จงแสดงว่า

$$\left (\frac{\sum_{j=1}^{n} \left (\prod_{k=1}^{j}a_k \right )^{\frac{1}{j}}}{\sum_{j=1}^{n}a_j} \right )^{\frac{1}{n}}+\frac{\left (\prod_{i=1}^{n}a_i \right )^{\frac{1}{n}}}{\sum_{j=1}^{n} \left (\prod_{k=1}^{j}a_k \right )^{\frac{1}{j}}}\le \frac{n+1}{n}$$

(China TST 2015)

3. จงหาจำนวนจริงบวก $\lambda$ ที่น้อยที่สุดที่มีสมบัติว่า

สำหรับจำนวนเชิงซ้อน $z_1,z_2,z_3$ ที่อยู่ภายในวงกลมหนึ่งหน่วย ถ้าหาก $z_1+z_2+z_3=0$ แล้ว

$$|z_1z_2+z_2z_3+z_3z_1|^2+|z_1z_2z_3|^2<\lambda$$

(China TST 2016)

4. ให้ $z_1,z_2,...,z_n$ เป็นจำนวนเชิงซ้อนและ $r\in (0,1)$โดยที่ $|z_i-1|\leq r$ จงแสดงว่า

$$|\sum_{i = 1}^{n} z_i||\sum_{i = 1}^{n} \frac{1}{z_i} |\geq n^2(1-r^2)$$

(China MO 2015 #1)

5. ให้ $x_1,x_2,...,x_n$ เป็นจำนวนจริงบวกโดยที่ $x_1+x_2+...+x_n=1$ จงแสดงว่า

$$(\sum_{i = 1}^{n} \frac{1}{1-x_i} )(\sum_{1\leq i<j\leq n} x_ix_j)\leq \frac{n}{2} $$

(China Western MO 2015 #3)
__________________
I'm Back

20 ตุลาคม 2016 23:22 : ข้อความนี้ถูกแก้ไขแล้ว 4 ครั้ง, ครั้งล่าสุดโดยคุณ Beatmania
ตอบพร้อมอ้างอิงข้อความนี้